Diễn Đàn MathScopeDiễn Đàn MathScope
  Diễn Đàn MathScope
Ghi Danh Hỏi/Ðáp Thành Viên Social Groups Lịch Ðánh Dấu Ðã Ðọc

Go Back   Diễn Đàn MathScope > Sơ Cấp > Tài Liệu > Đề Thi > Đề Thi HSG Cấp Tỉnh ở Việt Nam

News & Announcements

Ngoài một số quy định đã được nêu trong phần Quy định của Ghi Danh , mọi người tranh thủ bỏ ra 5 phút để đọc thêm một số Quy định sau để khỏi bị treo nick ở MathScope nhé !

* Nội quy MathScope.Org

* Một số quy định chung !

* Quy định về việc viết bài trong diễn đàn MathScope

* Nếu bạn muốn gia nhập đội ngũ BQT thì vui lòng tham gia tại đây

* Những câu hỏi thường gặp

* Về việc viết bài trong Box Đại học và Sau đại học


Trả lời Gởi Ðề Tài Mới
 
Ðiều Chỉnh Xếp Bài
Old 10-10-2013, 10:58 AM   #46
pco
+Thành Viên+
 
pco's Avatar
 
Tham gia ngày: Dec 2011
Bài gởi: 528
Thanks: 560
Thanked 195 Times in 124 Posts
Bây giờ còn hình học là unsolve.
[RIGHT][I][B]Nguồn: MathScope.ORG[/B][/I][/RIGHT]
 
__________________
"People's dreams... will never end!" - Marshall D. Teach.
pco is offline   Trả Lời Với Trích Dẫn
Old 16-10-2013, 04:49 PM   #47
namdung
Administrator

 
Tham gia ngày: Feb 2009
Đến từ: Tp Hồ Chí Minh
Bài gởi: 1,343
Thanks: 209
Thanked 4,066 Times in 778 Posts
Gửi tin nhắn qua Yahoo chát tới namdung
Kể từ ngày khởi động 1/10 đến nay là 15 ngày và với sự tham gia tích cực của các thành viên Mathscope, các đề thi và lời giải đã được cập nhật liên tục.

Tuần vừa rồi cũng như tuần này đã có hàng loạt đơn vị thi như Hà Nội, Hải Phòng, Tp HCM, An Giang, Thái Bình, Ninh Bình, Bà Rịa Vũng Tàu, Đồng Nai ...

Vì vậy chúng ta cần tiếp tục cập nhật đợt 2 cho đợt bình luận này.

Tôi tiếp tục gửi phần 2 các đề Tổ hợp.
[RIGHT][I][B]Nguồn: MathScope.ORG[/B][/I][/RIGHT]
 
File Kèm Theo
Kiểu File : doc 2VMO2014_Tohop2.doc (36.0 KB, 245 lần tải)
namdung is offline   Trả Lời Với Trích Dẫn
The Following 7 Users Say Thank You to namdung For This Useful Post:
dung_toan78 (16-10-2013), hoangqnvip (17-10-2013), nguyentatthu (17-10-2013), pco (16-10-2013), son1980 (22-10-2013), thaygiaocht (17-10-2013), vinh7aa (17-10-2013)
Old 17-10-2013, 06:14 PM   #48
luxubuhl
+Thành Viên+
 
Tham gia ngày: Nov 2011
Bài gởi: 253
Thanks: 115
Thanked 121 Times in 63 Posts
Phần PTH + Đa thức, phần này ít nên em xin post luôn ở đây ạ. Tổng hợp vào lại mất công đao về mở ra lại được 2 bài

Bài 1 ( Chọn ĐTQG TPHCM Ngày 1) Tìm $f:\mathbb{R} \to \mathbb{R}$ thỏa $$f(2013f(x+y)) = f(x+y) +2013f(x)f(y) - \frac{xy}{2013}$$



Bài 2 ( Chọn ĐTQG TPHCM ): Tìm các đa thức $f(x), g(x)$ hệ số nguyên thỏa $$f(g(x)) = x^{2013}+2014x+1 \ \forall x \in \mathbb{R}$$

Unsolved.

Bài 3 ( Chọn ĐTQG Phú Thọ Ngày 2): Tìm tất cả các đa thức $P(x);Q(x)$ hệ số thực, hệ số của số hạng chứ mũ cao nhất $=1$ và thỏa mãn $$P(1)+...+P(n)=Q(1+2+...+n)$$

Giải


[RIGHT][I][B]Nguồn: MathScope.ORG[/B][/I][/RIGHT]
 
luxubuhl is offline   Trả Lời Với Trích Dẫn
The Following 2 Users Say Thank You to luxubuhl For This Useful Post:
namdung (23-10-2013), nguyentatthu (23-10-2013)
Old 22-10-2013, 07:56 PM   #49
luxubuhl
+Thành Viên+
 
Tham gia ngày: Nov 2011
Bài gởi: 253
Thanks: 115
Thanked 121 Times in 63 Posts
Update một số bài toán số học mới.

Mọi người giải nhiệt tình lên nhé

Bài 1: Cho $p$ nguyên tố và $p\equiv3\pmod{4}.$ Hãy tìm số dư của phép chia $(1^2+1)(2^2+1)\cdots((p-1)^2+1)$ cho $p.$

Chọn ĐTQG Hải Phòng

Bài 2: Tìm số nguyên dương n nhỏ nhất sao cho $3^{n}-1$ chia hết cho $2^{2013}$

Chọn ĐTQG Bắc Ninh

Bài 3: Cho dãy số nguyên $(a_n)$ xác định như sau :
$\left\{\begin{matrix} a_0=1,a_1=3,a_2=5\\ a_{n+3}=2a_{n+2}+2a_{n+1}-a_n \end{matrix}\right.$
Tìm số nguyên $k$ sao cho $4a_na_{n+1}+k$ là số chính phương với mọi $n$ nguyên dương

Chọn ĐTQG Thái Bình

Bài 4: Cho $p$ là số nguyên tố lẻ. Tìm tất cả hàm số $f: \mathbb{Z}\to\mathbb{Z}$ sao cho với mọi $m,n\in \mathbb{Z}$ ta có :
1, Nếu $m\equiv n\pmod{p}$ thì $f(m)=f(n)$
2, $f(mn)=f(m)f(n)$

Chọn ĐTQG Thái Bình

Bài 6: Tìm tất cả các số tự nhiên $n$ sao hco $3^n+5$ là số chính phương.

Chọn ĐTQG Khánh Hòa

Bài 7: Cho 10 số nguyên dương $a_1, a_2,..... a_10$. Chứng minh rằng tồn tại các số $x_i \in \begin{bmatrix} -1;0;1 \end{bmatrix}$ không đồng thời bằng không với $i=1,2,....,10$ sao cho số $\sum_{i=1}^{10}x_ia_i \vdots 1023$

Chọn đội tuyển QG tỉnh Yên Bái 2013-2014 (Vòng 2)


____

Trong đợt 1 mình có tổng hợp và convert lại file số học sang pdf cho đẹp, tiện chém gió + thêm vài bài tương tự cho nó dài

Mọi người có thể tham khảo.

Ai có nhiều thời gian thì có thể tổng hợp tất cả các lời giải vào một # để mình làm thành file cho dễ. Hoặc bạn nào rảnh có thể pm mình mình đưa file text cho.
[RIGHT][I][B]Nguồn: MathScope.ORG[/B][/I][/RIGHT]
 
File Kèm Theo
Kiểu File : pdf số học.pdf (165.4 KB, 380 lần tải)

thay đổi nội dung bởi: luxubuhl, 22-10-2013 lúc 09:35 PM
luxubuhl is offline   Trả Lời Với Trích Dẫn
The Following 3 Users Say Thank You to luxubuhl For This Useful Post:
CôngNguyễn LTV (23-10-2013), namdung (23-10-2013), nguyentatthu (23-10-2013)
Old 23-10-2013, 01:53 PM   #50
namdung
Administrator

 
Tham gia ngày: Feb 2009
Đến từ: Tp Hồ Chí Minh
Bài gởi: 1,343
Thanks: 209
Thanked 4,066 Times in 778 Posts
Gửi tin nhắn qua Yahoo chát tới namdung
Trích:
Nguyên văn bởi luxubuhl View Post
Bài 2 ( Chọn ĐTQG TPHCM ): Tìm các đa thức $f(x), g(x)$ hệ số nguyên thỏa $$f(g(x)) = x^{2013}+2014x+1 \ \forall x \in \mathbb{R}$$

Unsolved.
Bài này đã được Lê Phúc Lữ giải triệt để trong topic Thi chọn đội tuyển Tp HCM.
[RIGHT][I][B]Nguồn: MathScope.ORG[/B][/I][/RIGHT]
 
namdung is offline   Trả Lời Với Trích Dẫn
The Following 2 Users Say Thank You to namdung For This Useful Post:
king_math96 (30-10-2013), tangchauphong (30-10-2013)
Old 11-11-2013, 08:42 PM   #51
linh1997
+Thành Viên+
 
Tham gia ngày: Nov 2011
Bài gởi: 133
Thanks: 27
Thanked 31 Times in 15 Posts
Bổ sung thêm 2 bài tổ hợp vòng 2 khtn nha!

Câu 2:
Cho m, n nguyên dương , S = { 1, 2, ... , 2014 } . Tìm số tập con của S có m số chẵn, n số lẻ và không chứa 2 số liên tiếp.
Bài 4: Có tồn tại hay không một tập hữu hạn các điểm xanh và đỏ trong mặt phẳng sao cho với mọi đường tròn đơn vị có tâm là một điểm xanh đều có đúng 10 điểm đỏ, và số điểm xanh nhiều hơn số điểm đỏ
[RIGHT][I][B]Nguồn: MathScope.ORG[/B][/I][/RIGHT]
 
__________________
lúc khó khăn nhất là lúc thành công không còn xa nữa
linh1997 is offline   Trả Lời Với Trích Dẫn
Old 16-11-2013, 09:40 PM   #52
quocbaoct10
+Thành Viên Danh Dự+
 
quocbaoct10's Avatar
 
Tham gia ngày: Oct 2012
Đến từ: THPT chuyên Lê Quý Đôn-Nha Trang-Khánh Hòa
Bài gởi: 539
Thanks: 292
Thanked 365 Times in 217 Posts
Em xin tổng hợp 1 số lời giải cho phần tổ hợp:
12. chuyển bài toán về dạng hình học, 18 người biểu diễn thành 18 điểm, nối các điểm này lại với nhau. Đoạn thẳng được tô màu xanh nếu như 2 người ở hai đầu mút quen nhau và mau đỏ nếu họ không quen nhau. Ta sẽ chứng minh tồn tại 1 từ giác mà các cạnh và đường chéo của nó được tô cùng màu.
Chứng minh: Một điểm M được nối với 17 điểm còn lại tạo thành 17 đoạn thẳng, theo nguyên lí đi-rích-lê, tồn tại 9 cạnh cùng màu, giả sử là màu xanh.
Xét 9 điểm khác M ở đầu mút các đoạn thẳng xanh kẻ từ M.
Nếu tồn tại tam giác nào có 3 cạnh cùng màu xanh thì tam giác đó tạo với đỉnh M 1 tứ giác thỏa ycđb.
Nếu không có tam giác nào có 3 cạnh cùng mau xanh:
Nếu tồn tại điểm A nào đó sao cho từ điểm đó xuất phát ít nhất 4 đoạn htanwgr màu xanh (giả sử là AB,AC,AD,AE) thì bài toán được chứng minh.
Nếu điểm nào trong 9 điểm cũng chỉ là đầu mút của nhiều nhất 3 đoạn xanh, ta thấy rằng không thể xảy ra trường hợp cả 9 điểm đều là đầu mút của cả 3 đoạn xanh vì như thế thì số đoạn thẳng xanh là $\frac{9 \times 3}{2}$ không là số nguyên.
Như vậy tồn tại ít nhất 1 điểm sao cho nó là đầu mút của nhiều nhất 2 đoạn thẳng xanh, đồng nghĩa với nó là đầu mút của 6 đoạn đỏ.
Giả sử 6 đoạn đó là AB,AC,AD,AE,AF,AG.
THeo bài toán quen thuộc thì ta luôn có 3 tam giác có 3 cạnh cùng màu, giả sử là BCD mà tam giác nào cũng phải có ít nhất 1 đoạn đỏ nên BCD có 3 cạnh màu đó từ đó suy ra ABCD là tứ giác thỏa ycđb.
14.Trước hết, ta sẽ chứng minh rằng trò chơi sẽ kết thúc sau hữu hạn lần.

Thật vậy, ta coi mỗi bóng đèn sáng là 1 và mỗi bóng đèn tắt là 0. Khi đó, dãy bóng đèn như thế sẽ tương ứng với một xâu nhị phân có giá trị nhất định trong hệ thập phân.
Giá trị của xâu nhị phân ban đầu là $S = 2^{2014}-1$. Sau mỗi lần thay đổi, người ta chọn 5 số liên tiếp mà bắt đầu bởi 1 rồi đổi tương ứng 1 thành 0, 0 thành 1, nghĩa là đã làm giảm giá trị của xâu nhị phân này, từ $S$ thành $S'$ mà $S' < S$ rồi cứ như thế.
Dễ thấy rằng nếu giảm đến khi toàn bộ 2010 bit đầu tiên trở thành 0 thì không thể thực hiện tiếp thao tác nào được nữa nên số nhỏ nhất mà người chơi vẫn có thể thao tác được là $10000_2=32$.
Rõ ràng $S-32$ là hữu hạn nên trò chơi sẽ được tiến hành trong không quá $S-32$ lần.

Tiếp theo, để chứng minh rằng người thứ nhất luôn thua. Ta có nhận xét sau:
Trong 2010 bóng đèn phía trước, ban đầu có số bóng đèn sáng là chẵn.
Sau mỗi lượt thao tác, ít nhất một trong các bóng đèn này sẽ thay đổi trạng thái (vì nếu không thì chỉ có 4 bóng sau bị thay đổi trạng thái, vô lí).
Với 5 bit bất kì liên tiếp, giả sử có $x$ bit là 1, $5-x$ bit là 0 thì sau khi thao tác, ta có $5-x$ bit là 1, $x$ bit là 0. Số bit 1 thay đổi từ $x$ thành $5-x$, tức là một số lẻ đơn vị.

Trạng thái thua xuất hiện khi toàn bộ 2010 bit ban đầu trở thành 0, tức là chuyển 2010 bit 1 thành 0 bit 1, theo trên thì nó đòi hỏi một số chẵn lần. Điều này có nghĩa là người 1 không thể đạt được trạng thái đó.

Do trò chơi sẽ kết thúc sau một số lần hữu hạn nên người 1 luôn thua trong trò chơi này.

Tổng quát lên, ta có thể thấy rằng với các số chẵn lớn hơn 4 thì người thứ nhất luôn thua, với các số lẻ lớn hơn 4 thì người thứ hai luôn thua.
15.Trong bảng này có $m$ hàng và $n$ cột với $m < n$ nên nếu ta đặt $a_1, a_2, ..., a_m$ là số các viên bi ở các hàng và $b_1, b_2, ..., b_n$ là số các viên bi ở các cột thì theo giả thiết, ta có:
$b_i > 0, i = 1,2,3,...,n$.
Rõ ràng $a_1+a_2 + ... + a_m = b_1 +b_2+ ...+ b_n = S$.
Tổng của $m$ số ở VT bằng tổng của $n$ số ở VP, mà $m < n$ nên phải tồn tại một số ở VT lớn hơn VP. Giả sử $a_i > b_j$, khi đó số ở ô $(i,j)$ thỏa mãn đề bài.

Ta sẽ chứng minh rằng ngoài ô này ra, còn có một ô khác nữa.
Thật vậy, nếu như ngoài cặp $(a_i, b_j)$, còn lại ta đều có $a_r \le b_s$ với $1 \le r \le m, 1 \le s \le n$ và $(r,s) \neq (i,j)$.
Do $n > m$ nên $n \ge m+1$, suy ra $n- (m-1) \ge 2$.
Ta thấy rằng $a_i = S - k$ với $k$ là tổng của $m-1$ số khác $a_i$ ở VT. Chọn ra trong VP, $m-1$ số bất kì có chứa $b_j$ trong đó thì rõ ràng tổng của $m-1$ số này là $k'$ với $k' \ge k$, còn lại ít nhất 2 số $b_x, b_y$ nào đó mà $b_x, b_y \ge a_i$.
Suy ra $a_i +k \ge b_x + b_y + k' \ge 2a_i + k$ hay $a_i \le 0$, vô lí.

Vậy phải tồn tại ít nhất 2 viên bi mà hàng chứa nó có nhiều bi hơn cột chứa nó.
16. ta đếm số bộ $(x_1,x_2,...,x_i)$ thỏa $x_1=a$. Với mỗi $x_i , i=2,3 ... n$ thì có 2 cách chọn nên số bộ này có $2^{n-1}$ bộ. đặt $a_n$ là số dãy thỏa $x_1=1$ và $x_n=a$; $b_n$ là số dãy thỏa $x_n=b$ và $x_1=1$, $c_n$ là số dãy thỏa $x_n=c$ và $x_1=1$, khi đó dễ có ctth: $a_{n+1}=b_n+c_n$ và $a_n+b_n+c_n=2^{n-1}$ ($n \ge 3$). Từ đây, ta được ctth:$a_n+a_{n+1}=2^{n-1}$ với $a_n$ là số dãy thỏa ycđb, $n \ge 3$.
18.gọi $x_i$ ($1 \le i \le n$) là tọa độ các bạn nam trên trục số, $y_i$ ($1\ le i \le n$) là tọa độ các bạn nữ tren trục số.
đặt $A_n$ là tổng các khoảng cách giữa các bạn cùng giới tính, $B_n$ là tổng khoảng cách giữa các bạn khác giới.
$x_1<x_2<...<x_n$, tương tự với các $y_i$.
ta sẽ chứng minh khẳng định bằng quy nạp.
Dễ thấy $n=1$ đúng.
Giả sử $B_{n-1} \ge A_{n-1}$
$A_n-A_{n-1}=\sum_{i=1}^{n-1}[(x_n-x_i)+(y_n-y_i)]=\sum_{i=1}^{n-1} [x_n-y_i+y_n-x_i]$
$B_n-B_{n-1}=\sum_{i=1}^{n-1}[|x_n-y_i|+|y_n-x_i|]+|x_n-y_n|$.
Từ đó suy ra $B_n-B_{n-1} \ge A_n-A_{n-1}$. Từ đây kết hợp với giả thiết quy nạp, ta có đpcm.
19.Gọi $M$ là tập các số có dạng $4^n.k$ trong $S$, $n$ không âm và $k$ lẻ. Ta đếm được $|M|=2013$ (cho $n$ chạy từ $1 \to 6$ rồi đếm $k$).
Gọi $S_x = S \bigcap \{ x; 2x\}\ \forall x \in M$ và $\displaystyle S' = \bigcup_{x \in M} S_x$.
Nhận xét rằng $M$ đã chứa các số dạng $2^{2n}.k$ và do đó $S'$ là tập M bổ sung thêm các số dạng $2^{2n+1}.k$. Suy ra $S' = S$.

Vậy có thể phân hoạch tập $S$ thành 2013 tập (dễ thấy là không giao nhau) $S_x \ \ (x \in M)$.
Do đó, theo nguyên lý Dirichlet, 1 tập con bất kì của S có 2014 phần tử sẽ chứa 1 tập $S_x$ và do đó không thỏa giả thiết.

[RIGHT][I][B]Nguồn: MathScope.ORG[/B][/I][/RIGHT]
 
__________________
i'll try my best.

thay đổi nội dung bởi: quocbaoct10, 16-11-2013 lúc 09:47 PM
quocbaoct10 is offline   Trả Lời Với Trích Dẫn
The Following 2 Users Say Thank You to quocbaoct10 For This Useful Post:
hoangqnvip (17-11-2013), phatsp (05-05-2014)
Trả lời Gởi Ðề Tài Mới

Bookmarks

Ðiều Chỉnh
Xếp Bài

Quuyền Hạn Của Bạn
You may not post new threads
You may not post replies
You may not post attachments
You may not edit your posts

BB code is Mở
Smilies đang Mở
[IMG] đang Mở
HTML đang Tắt

Chuyển đến


Múi giờ GMT. Hiện tại là 11:38 AM.


Powered by: vBulletin Copyright ©2000-2024, Jelsoft Enterprises Ltd.
Inactive Reminders By mathscope.org
[page compression: 82.70 k/91.47 k (9.59%)]